http://7sage.com/lsat_explanations/lsat-56-section-3-question-20/
The ... (D) without hesitation. The question stem asks "which one of ... heart disease. Furthermore, the question stem asks for "most weaken ... to (B) given the question stem?
http://7sage.com/lsat_explanations/lsat-26-section-3-question-24/
This question is beyond sucky. Did anyone else have any problems with determining the right answer for this question? Thanks.
... ran across a question in particular (LSAT 58, Section 4, Question 21) regarding lottery ... to it: will an Assumption Question answer choice ever feature a ... more detailed eye? Hope this question makes sense, and I look ...